hshsjshshsjsksjjsbsbbs bc hd​

Hshsjshshsjsksjjsbsbbs Bc Hd

Answers

Answer 1

Answer:

its b b b

Step-by-step explanation:


Related Questions

Hello, Brainly community!

This question is for all of those Calculus people out there.

The volume of a swimming pool is changing with respect to time, such that the volume is given by W(t), where W(t) is measured in cubic centimeters and t is measured in seconds. A tangent line is shown for W(t) at t = 3 seconds. Determine the best estimate for the value of the instantaneous rate of change of W(t) when t = 3.
(I've narrowed down the answer choices to 2, and just really need to find the right way of thinking to find the answer)

(A) W(lim t) as t goes to 3.
(B) [W(3.1) - W(2.9)] / 0.2.

Thank you in advance!

Answers

Answer:

(B)  [tex]\displaystyle \frac{W(3.1) - W(2.9)}{0.2}[/tex]

General Formulas and Concepts:

Calculus

Limits

Derivatives

The definition of a derivative is the slope of the tangent line.

Derivative Notation

Instantaneous Rates

Tangent Line: [tex]\displaystyle f'(x) = \frac{f(b) - f(a)}{b - a}[/tex]

Step-by-step explanation:

Since we are trying to find a rate at which W(t) changes, we must find the derivative at t = 3.

We are given 2 close answer choices that would have the same numerical answer but different meanings:

(A)  [tex]\displaystyle \lim_{t \to 3} W(t)[/tex](B)  [tex]\displaystyle \frac{W(3.1) - W(2.9)}{0.2}[/tex]

If we look at answer choice (A), we see that our units would simply just be volume. It would not have the units of a rate of change. Yes, it may be the closest numerically correct answer, but it does not tell us the rate at which the volume would be changing and it is not a derivative.

If we look at answer choice (B), we see that our units would be cm³/s, and that is most certainly a rate of change. Answer choice (B) is also a derivative at t = 3, and a derivative tells us what rate something is changing.

∴ Answer choice (B) will give us the best estimate for the value of the instantaneous rate of change of W(t) when t = 3.

Topic: AP Calculus AB/BC (Calculus I/I + II)

Unit: Differentiation

Book: College Calculus 10e

What is the scale of the y-axis in this coordinate graph?


A. 1 tick mark represents 1 unit
B. 1 tick mark represents 8 units
C. 1 tick mark represents 12 units
D. 1 tick mark represents 16 units

Answers

Answer:

Obviously B

Step-by-step explanation:

2 1/4 x 3 1/5 brainliest

Answers

Answer:

36/5

Step-by-step explanation:

9/4×16/5

144/20

36/5

hope this is helpful

Answer:

[tex]7\frac{1}{5}[/tex]

Step-by-step explanation:

1. start by turning the fractions improper fractions:

[tex]2\frac{1}{4} =\frac{9}{4}[/tex]

[tex]3\frac{1}{5} =\frac{16}{5}[/tex]

2. then multiply them together:

[tex]\frac{9}{4}[/tex] x [tex]\frac{16}{5}[/tex] = [tex]\frac{144}{20}[/tex]

3. then simplify the fraction:

[tex]\frac{144}{20}[/tex][tex]=\frac{36}{5}[/tex]

4. turn it into a proper fraction:

[tex]\frac{36}{5} =7\frac{1}{5}[/tex]

Nathan is 1.55 meters tall. At 1 p.m., he measures the length of a tree's shadow to be 38.15 meters. He stands 32.9 meters away from the tree, so that the tip of his shadow meets the tip of the tree's shadow. Find the height of the tree to the nearest hundredth of a meter .

Answers

Answer:

11.26 m

Step-by-step explanation:

The height of the tree is about 11.25 meters.

What are similar triangles?

When the respective sides are proportional and the corresponding angles are congruent, two triangles are said to be similar.

Given that, the height of the person is 1.55 meters, the length of the tree's shadow is 38.15 meters, and the distance between the person and the tree is 32.9 meters.

Let the height of the tree be x.

Note that the scenario makes two similar triangles.

Since the ratio of the side lengths of similar triangles is proportional, it follows:

(38.15 - 32.9)/1.55 = 38.15/x

5.25/1.55 = 38.15/x

3.39 = 38.15/x

x = 38.15/3.39

x = 11.25

Hence, the height of the tree is about 11.25 meters.

Learn more about similar triangles: https://brainly.com/question/25882965

#SPJ2

5) On each birthday Rosa gets as many roses as she is old in years. She still has all the dried flowers and there are now 120 of them. How old is she? A) 10 B) 12 C) 14 D) 15 E) 20

Answers

Answer:

D) 15

Step-by-step explanation:

This is an arithmatic progression.

The formula for the sum of arithmatic progression is

[tex]s = \frac{n}{2} (2a + (n - 1)d)[/tex]

where d is the common difference between successive terms and a is the first term. By applying this formula,

[tex]120 = \frac{n}{2} (2(1) + (n - 1)(1)) \\ 120 = \frac{n}{2} (1 + n) \\ n(1 + n) = 240 \\ n {}^{2} + n - 240 = 0 \\ (n - 15)(n + 16) = 0 \\ n = 15 \: or \: n = - 16(reject)[/tex]

The cost for 2 adults and 5 children to eat at the local buffet restaurant is \$46.50 . The cost for 1 adult and 1 child is $15. What is the cost of a child's meal ?

Answers

Answer:

c = 5.5

Step-by-step explanation:

2 a + 5 c = 46.50

a + c = 15

a = 15 - c

2(15-c ) +5c = 46.50

30 - 2c +5c = 46.5

3c = 16.5

c = 5.5

Let a be the adults and c be the children.

For the first equation there are 2 adults and 5 children so the equation is:-

2a + 5c = 46.50

And for the second equation there is 1 adult and 1 child. So the equation is:-

a + c = 15

a = 15 - c

You take this formula and substitute it to the above formula (2a + 5c = 46.50)

➡️ 2a + 5c = 46.50

➡️ 2(15 - c) + 5c = 46.50

➡️ 30 -2c + 5c = 46.50

➡️ 5c - 2c = 46.50 - 30
(You collect like terms)

➡️ 3c = 16.5

➡️ c = 16.5/3

➡️ c = 5.5

help and explain//////////////////////////////////////////////

Answers

Answer:

(f + g)(x) = 5x - 3

Step-by-step explanation:

(f + g)(x)

= f(x) + g(x)

= 2x + 4 + 3x - 7 ← collect like terms

= 5x - 3

Answer:

5x - 3

Step-by-step explanation:

(f + g)(x) = f(x) + g(x)

= 2x + 4 + ( 3x - 7 )

= 5x - 3

Heya Kitties!
What is the value of x?

18x−16=−12x−4



x =

Answers

Answer:

2/5

Step-by-step explanation:

18x+12x=-4+16

30x=12

x=2/5

Answer:

18x-16=-12x-4

18x+12x=-4+16

30x=12

x=12/30

x=2/5

Select the expression that represents the following statement: add 24 to the quotient of 16 and 8.

Answers

Answer:

16/8 + 24

Step-by-step explanation:

If a bus travel for 120 minutes at a speed of 75 kilometers per hour how far has the bus traveled?

Answers

Answer:

150 km

Step-by-step explanation:

Put the minutes into hours 120min is 2 hours.

Distance = speed * time

Distance = 75 * 2

Distance = 150

Answer:

150 kilometers

Step-by-step explanation:

if the bus is going 75 kilometers an hour and they traveled for 120 minutes (exactly 2 hours) then you would just multiply 75 by 2 to get 150 kilometers total.

Evaluate 2y when y = 6y

Answers

I believe the answer is 12y
im sure the answer is 12y

There are 35 times as many students at Wow University as teachers. When all the students and
teachers are seated in the 8544 seat auditorium, 12 seats are empty. How many students attend
Wow University?

Answers

Given:

There are 35 times as many students at Wow University as teachers.

When all the students and teachers are seated in the 8544 seat auditorium, 12 seats are empty.

To find:

The total number of students.

Solution:

Let x be the number of teachers at Wow University. So, the number of student is :

[tex]35\times x=35x[/tex]

When all the students and teachers are seated in the 8544 seat auditorium, 12 seats are empty.

[tex]x+35x=8544-12[/tex]

[tex]36x=8532[/tex]

[tex]x=\dfrac{8532}{36}[/tex]

[tex]x=237[/tex]

The number of total students is:

[tex]35x=35(237)[/tex]

[tex]35x=8295[/tex]

Therefore, the total number of students is 8295.

Decrease £19064.67 by 9.5%

Give your answer rounded to 2 DP. (decimal place)

Please Help !!!

Answers

Answer:

17253.52

Step-by-step explanation:

100-9.5=90.5

19064.67*0.905=17253.52

Find value of x.
A. 110
B. 47
C. 68
D. 112

Answers

Answer:

B

Step-by-step explanation:

The sum of the inner angles of a quadrilateral is 360 degrees

135 + 110 + 68 + x = 360

313 + x = 360

x = 47 degrees

Answer:

47

Step-by-step explanation:

whole thing is 360 degrees

68 + 110 + 135 = 313

360 - 313 = 47

x looks small too (if you had to guess in a multiple choice question)

pls help me don't know what to do

Answers

Answer:

x=15

Step-by-step explanation:

The 60 degree angle and the (x+45) degree angle are both the same degree because they are vertical angles.

So to solve, just subtract 45 from 60

60-45=15

That's your answer!

Hope this helps!

solve for x. Round to the nearest tenth, if necessary.​

Answers

Answer:

7.1

Step-by-step explanation:

We used SOHCAHTOA because it's a right angle triangle

So because we have an angle with an adjacent of 6.3 and hypotenuse of x

We will use

Cos=adjacent /hypotenuse

7.1 like the last answer if not you can round that

The area of the rectangular sandbox at Dave's school is 117 square feet. The sandbox has a width of 9 feet as shown in the diagram. What is the perimeter of the sandbox?

Answers

Answer:

ay bru ima tell yu dhis rn is c

Step-by-step explanation:

The center of the circle is at the point
, and its radius is
units. The equation of this circle in standard form is
.

Answers

Is there a picture that goes with this?

D=22/7×d-90 Solve the equation
Find D​
Fast!

Answers

Answer:

D=22-90+22d/7

Step-by-step explanation:

D=22/7×d-90

D=-90+22d/7

Answer is- d=-630/-22x+7

The formula for the circumference of a circle is R = c/2(pi)

Find the radius of a circle that has a circumference of 16(pi)

A) r = 4
B) r = 8
C) r = 12
D) r = 16

Answers

The answer is B) r= 8

What type of health screening would this patient most likely receive?
Sue is a 45-year-old woman with a family history of breast cancer. Her healthcare professional will most likely recommend that she receive a .

Answers

Answer:

A mammogram is what she would receive

Step-by-step explanation:

Choose the smallest number 3 1/8 or 10/3

Answers

Answer:

10/3

Step-by-step explanation:

31/8= 3.8

10/3= 3.3

find the slope of the line passing through the points (-2,5) and (3/2,2)

Answers

Answer:

slope = - [tex]\frac{6}{7}[/tex]

Step-by-step explanation:

Calculate the slope m using the slope formula

m = [tex]\frac{y_{2}-y_{1} }{x_{2}-x_{1} }[/tex]

with (x₁, y₁ ) = (- 2, 5) and (x₂, y₂ ) = ([tex]\frac{3}{2}[/tex], 2)

m = [tex]\frac{2-5}{\frac{3}{2}-(-2) }[/tex]

   = [tex]\frac{-3}{\frac{3}{2}+2 }[/tex]

   = [tex]\frac{-3}{\frac{7}{2} }[/tex]

  = - 3  × [tex]\frac{2}{7}[/tex]

  = - [tex]\frac{6}{7}[/tex]

According to the number line, what is the distance between points A and B?

0 6 units
7 units
O 12 units
O 14 units

Answers

Answer:

14 units

Step-by-step explanation:

A = - 2, B = 12

Therefore,

d(A, B) = 12 - (-2) = 12 + 2 = 14 units

PLS HELP SOON WILL MARK BRAINLYEST

A railroad tunnel is shaped like a semi-ellipse, as shown below. A semiellipse is shown on the coordinate plane with vertices on the x axis and one point of intersection with the positive y axis. The height of the tunnel at the center is 35 ft, and the vertical clearance must be 21 ft at a point 8 ft from the center. Find an equation for the ellipse.

Answers

According to the question

b= 35 and (8,21) lies on the ellipse

After calculation we get a= 10

equation for the ellipse.

[tex] \frac{ {x}^{2} }{100} + \frac{ {y}^{2} }{1225} = 1[/tex]

Which operation will solve the following word problem? Jeff earns $14.00 per hour, Tom earns half as much as Jeff. How much does Tom earn per hour?


Multiplication


Subtraction


Addition


Division

Answers

Answer:

The correct option is (d).

Step-by-step explanation:

Given that,

Jeff earns $14.00 per hour.

Tom earns half as much as Jeff.

We need to find the amount earn by Tom per hour.

Tom's amount = Jeff's amount/2

So,

[tex]T=\dfrac{14}{2}\\\\T=\$7[/tex]

So, Tom earn $7 per hour. Hence, division operation is used. Jeff's amount is divided by 2.

On the unit circle, which of the following angles has the terminal point
coordinates.
A. 45
B. 135
C. 225
D. 315

Answers

Answer: C. 225

Step-by-step explanation:

jordan wants to save to buy a car and decides to open a banking account that is offering an interest rate of 4.5% compounded annually how much will jordan have in the account after 5 years it he deposits $7,000 today?

Answers

Answer:

8,723.27$

Step-by-step explanation:

find the domain of f(x)=sec(2x)

Answers

Answer:

*Refer the image attached

Step-by-step explanation:

*Refer the image attached

A cinema is doing a promotion to celebrate their 50th anniversary for 1 week. They give

away a free drink to every 98th customer, a free bag of popcorn to every 112th customer and

a free cinema ticket to every 224th customer. Which lucky customer will be the first to

receive all 3 items?​

Answers

Answer:

1,568 customer

Step-by-step explanation:

Find the lowest common multiple of 98, 112, and 224

98 = 98, 196, 294, 392, 490, 588, 686, 784, 882, 980, 1078, 1176, 1274, 1372, 1470, 1568, 1666

112 = 112, 224, 336, 448, 560, 672, 784, 896, 1008, 1120, 1232, 1344, 1456, 1568, 1680, 1792, 1904

224 = 224, 448, 672, 896, 1120, 1344, 1568, 1792, 2016, 2240

The lowest common multiple of 98, 112, and 224 is 1568

Therefore, the 1,568th customer will be the first to receive all 3 iitem

Other Questions
An FI purchases at par value a $100,000 Treasury bond paying 10 percent interest with a 7.5 year duration. If interest rates rise by 4 percent, calculate the bond's new value. Recall that Treasury bonds pay interest semiannually. Use the modified duration valuation equation. A retail department store is approximately square, 35 meters (100 feet) on each side. Each wall has two entrances equally spaced apart. Located at each entrance is a point-of-sale cash register. Suggest a local area network solution that interconnects all eight cash registers. Draw a diagram showing the room, the location of all cash registers, the wiring, the switches, and the server. What type of wiring would you suggest? autobiographical meaning Find the perimeter of a football field which measures 90m by 60m ILOVEYOU SO MUCH SILVIA, NO MATTER WHAT HAPPEN in the ____ paragraph you request action, such as an appointment, an interview, or some other definite action What is the theme of Ovids The Story of Daedalus and lcarus? A 5.0 kg block of ice is at rest at the top of a smooth inclined plane. The block is released and slides 2.0 m down the plane. Assuming there is no friction between the block and the surface, calculatea) the gravitational potential energy at the top of the planeb) the component of the weight parallel to the plane c) the acceleration of the block d) the velocity of the block at the bottom of the plane e) the kinetic energy at the bottom of the plane. 4019K Express your answer as a nuclear equation. hlw guys plz help me which set is this.for examples: A u B , A u B u C...like that..plz help me write down the following units in the ascending of their value A) mm nm cm um B) 1m 1cm 1km 1mm. convert the following units into SI without changing their values? A)3500g B)2.5km C)2h B.Fill in the blanka with the correct word given inside the box.clay. water. sticky. loamoxygen. sand. plants. particles7.________ is made up of particles that are tightly packed together.Very fine 8._________ of rocks can hold much water and become 9._________when wet.While 10.________ is a mixture of sand and clay,this contains the right mixture of particles of silt,sand and clay. TIMED HELP PLEASE. Determine whether the equation is an identity or not an identity. X,Y and Z from a business with capitals Rs 5000,Rs.4500 and Rs.6500 respectively,after 6 month,X doubles has capital and after next 3 months Y trebles his capital .If the profit at the end of the year amount to RS.8300,find the profit obtained by each X,Y and Z. He's very short: ________ sisters are taller. (A) both of them (B) his both (C) both his (D) the two both his Nine children are to be divided into an A team, a B team and a C team of 3 each. The A team will play in one league, the B team in another, the C team in a third league. How many different divisions are possible If (3x 2)(3x + 2) = ax2 b, what is the value of a? What might Paul Laurence Dunbar mean in the following lines from his poem "We Wear the Mask"? Unit Test Unit Test Active 1 2 3 4 5 6 7 CO Given fix) = 17- x2, what is the average rate of change in f(x) over the interval [1, 5]? -6, -1/2, 1/4, 1 Which of the following is correct? 0 0 0 0 Please lie this book on the table. Please except my apology. He tried to affect our vote. I need to set down and rest awhile.